Hello every Brilliant Mind! This is the first season of Brilliant Sub Junior Number Theory Contest. This contest is for beginners or intermediate ones who want to sharpen their skills of problem-solving in overall Number Theory. .
The aim of this contest is to improve the skill of in the computation in all sorts of problem (of basic level) in Number Theory like Proof problems , Types of Congruences , Theorems related to Modular Arith. etc. by learning from each other and of course to have fun !
Eligibility
People should fulfill either of the 2 following:
16 years or below
Minimum level preference : Level 3 in Number Theory .
Eligible people here may participate in this contest.
:
(There is a slight change in the rules of the contest : Now there can be more(2-3) unsolved problems(they are called 'Extra Participation Problem(EPP)) ! , so many people can participate in contest at once ! :) One rule to be noted about these EPP's is that , those problems are to be posted until regular problem is unsolved.For each new unsolved problem , there will be new set of EPP's ! Happy Solving !)
I will start by posting the first problem. If there is a user solves it, then they must post a new one.
You may only post a solution of the problem below the thread of problem and post your proposed problem in a new thread. Put them separately.
Only make substantial comment that will contribute to the discussion.
Make sure you know how to solve your own problem before posting it in case there is no one can answer it within 48 hours, then you must post the solution and you have a right to post another problem.
If the one who solves the last problem does not post his/her own problem after solving it within a day, then the one who has a right to post a problem is the last solver before him/her.
It is NOT compulsory to post original problems. But make sure it has not been posted on brilliant..
Please post your solution and your proposed problem in a single new thread.
SOLUTION OF PROBLEM xxx (number of problem) :
[Post your solution here]
PROBLEM xxx (number of problem) :
[Post your problem here]]]
[Name of the solver]
The comments will be easiest to follow if you sort by "Newest":
Proofs related to congruences , Number Theoretic inequalities , Eulers Theorem , Modular Arithmetic , Number Theoretic Functions , Order of an Integer modulo .
Easy Math Editor
This discussion board is a place to discuss our Daily Challenges and the math and science related to those challenges. Explanations are more than just a solution — they should explain the steps and thinking strategies that you used to obtain the solution. Comments should further the discussion of math and science.
When posting on Brilliant:
*italics*
or_italics_
**bold**
or__bold__
paragraph 1
paragraph 2
[example link](https://brilliant.org)
> This is a quote
\(
...\)
or\[
...\]
to ensure proper formatting.2 \times 3
2^{34}
a_{i-1}
\frac{2}{3}
\sqrt{2}
\sum_{i=1}^3
\sin \theta
\boxed{123}
Comments
d1,d2,.....d2n can be from 1,2,3,4,...,9 only
j=1∑nd2j−1d2j is even If
(1) all the n terms are even
(2) the number of odd terms in the sum is even
d2j−1d2j is even if
(1) both d2j−1d2j are even . There are 16 choices for this
(2) one of them is odd the other one is even ; there are 2(5)(4)=40 choices for this d2j−1 could be odd and d2j even or vice versa ).
Note d2j−1d2j is odd for 25 choices .
For the sum to be even , an even number of such products must be odd . Number of ways in which k of the n terms are odd and the rest are even is (kn).25k(56)n−k.
Total sum is even if k′s are even .
Total number of ways
Pn=k=0∑⌊2n⌋(2kn)252k56n−2k
Here⌊.⌋ represents floor function.
Let Qn=k=0∑⌊2n⌋(2k+1n)252k+156n−2k−1
Pn+Qn=(56+25)n
Pn−Qn=(56−25)n
Hence Pn=281n+31n
Log in to reply
Nice solution.
Awesome ! nice solution ....
Post next regular problem ...
Problem 2
Prove that all numbers in the form 224n+1+7 and 226n+2+13 for n∈N are composite.
This problem has been solved by Julian Poon.
Log in to reply
Solution to Problem 2
Since the 2 questions are so similar, I'll just prove for one case.
From Euler's Theorem,
210n≡1 (mod11)
So,
224n+1 ≡2[24n+1 (mod 10)]mod 11
Using Euler's theorem again,
24n+1≡2 (mod 10)
Hence
2[24n+1 (mod 10)]=4 (mod 11)
The result follows, that 224n+1+7 is divisible by 11.
Similarly, you can prove that 226n+2+13 is divisible by 29.
@Akshat Sharda Can it be solved this way that all primes are of the form 6k + 1 or 6k - 1. Its easy to show that they are not of the either forms and hence they are composite???
Log in to reply
6(8)+1=49 and 6(6)-1=35
49 and 35 are composite.
Log in to reply
@Akshat Sharda what I meant is that all primes are of the form 6k +- 1 and not that all 6k+-1 are primes
Log in to reply
Log in to reply
@Akshat Sharda Leaving 2 and 3...I forgot to mention that....Btw Akshat you are in 9 or 10??
Log in to reply
No, because all numbers in the form above are -1, 1 mod 6
Log in to reply
@Julian Poon O sorry I didn't mark that the numbers above are of the form 6k-1
Problem 6
Find all positive integers (with proof) x,y,z such that 8x+15y=17z
This problem has been solved by Sharky Kesa.
Log in to reply
Here is my extremely long method to solving this question (First method I found) :P
8x+15y⇒1+1⇒17z⇒z =17z≡17z(mod7)≡2(mod7)≡2(mod6)
Thus, z is even. Also, we have
8x+15y8x0y=17z=17z−15y=1x−(−1)y(mod8)=0(mod2)
Thus, y is even. Also, we have (from looking at the equation in (mod10)), 84a+15b=174c and 84a+2+15b=174c+2. Thus, x is even. Let x=2x1, y=2y1 and z=2z1. Substituting back in, we get
26x126x1=172z1−152y1=(17z1−15y1)(17z1+15y1)
Because of FTA, we can let 17z1−15y1=2a and 17z1+15y1=2b, with a+b=6x1 and a<b. We now have
17z1−15y117z1+15y1⇒17y1=2a=2b=2b−1+2a−1
From this, since the LHS is odd, the RHS must be odd, so a=1 and b=6x1−1. Substituting this, we get
17z1−15y117z1+15y1⇒(−1)z1+0⇒z1=2=26x1−1≡−1(mod3)≡1(mod2)
17z1−15y117z1+15y1⇒1z1−(−1)y1⇒y1=2=26x1−1≡0(mod8)≡1(mod2)
Thus, y1 and z1 are odd, which implies that x1 is odd from 84a+2+15b=174c+2.
Going back to the original equation and substituting x=4x2+2, y=4y2+2 and z=4z2+2, we get
84x2+2+154y2+2=174z2+2154y2+2=(172z2+1−82x2+1)(172z2+1+82x2+1)
Once again, by FTA, let 172z2+1−82x2+1=3a5b and 172z2+1+82x2+1=3b5a, where a+b=4y2+2.
172z2+1−82x2+1172z2+1+82x2+1⇒(−1)2z2+1+(−1)2x2+1⇒(−1)−1=3a5b=3b5a≡0(mod3)≡0(mod3)
Since this isn't true, we have that b=0 so a=4y2+2. We now have
172z2+1−82x2+1172z2+1+82x2+1=92y2+1=252y2+1
For x2>0, we have 82x2+1≡0(mod64). Also, 172z2+1≡17,49(mod64) and 252y2+1≡25,9,57,41. Since both of these terms don't share any residues in (mod64), x2=0. Substituting this back in to the original equaition, we get
82+15y⇒17z1+15y117z1−15y1⇒17z1=1715y1=15⇒z1y1=17z=32=2=1=1
Thus, x=2, y=2, z=2 is the only solution.
Log in to reply
I think that if we work with modulo 32 or 64 the solution would be shorter. I haven't solved the problem yet but I think that there are some interesting results with it.
I have a similar (and equally long) solution and was hoping for a shorter one but... oh well. By the way, I think you have a typo where 1z1−(−1)y1≡0mod3 should be 1z1−(−1)y1≡0mod4
i have a very short solution to this question using Beals conjecture only if am permited to post it
Log in to reply
Are integers x,y,z are different?
Log in to reply
If they are not different , then first comes pythagoras XD
Problem 1
n=1∑Mp.(ϕ(1+pn−1))≤n=1∑Mpn+1−pn
Disprove (without giving examples , give a proof), for natural numbers M and p.
This problem has been solved by Akshat Sharda
Log in to reply
Solution to Problem 1
Let us consider,
n=1∑Mp.ϕ(1+pn−1)≤n=1∑Mpn+1−pnpn=1∑Mϕ(1+pn−1)≤(p−1)n=1∑Mpnpn=1∑Mϕ(1+pn−1)≤(p−1)⋅p⋅p−1pm−1n=1∑Mϕ(1+pn−1)≤pm−1
Now, as p∈N, for p=1,
n=1∑Mϕ(1+pn−1)=n=1∑Mϕ(1+1n−1)=M
And,
pM−1=1M−1=0
As M is a natural number it should be obviously more than 0, but inequality is giving us M≤0. Thus , it does not hold true.
Extra Participation Problem - Set 1 - #3
Let m,n∈N such that
lcm(m,n)+gcd(m,n)=m+n
Prove that one of the 2 numbers is divisible by the other.
This problem has been solved by Harsh Shrivastava.
Log in to reply
Let gcd(m,n)=k
Then LCM(m,n) = (mn)/k
(mn/k) + k =m+n
On rearranging, k2−(a+b)k+ab=0
Thus by quadratic formula, k = a or b
Hence proved.
Extra Participation Problem -Set 2: #2
Are n5+5 and (n+1)5+5 coprime for all positive integers n? If not, find all values of n for which this is not true.
Log in to reply
Here is my long, sort of bashy solution:
Suppose that d is a common factor of n5+5 and (n+1)5+5. Thus, we have the following:
d⇒d⇒d⇒d⇒d⇒d∣(n+1)5+5−(n5+5)=5n4+10n3+10n2+5n+1∣(5(n5+5)−(n−2)(5n4+10n3+10n2+5n+1)=10n3+15n2+9n+27∣4(5n4+10n3+10n2+5n+1)−(2n−1)(10n3+15n2+9n+27)=7n2−43n−23∣98(10n3+15n2+9n+27)−(140n+1070)(7n2−43n−23)=50112n+27256∣1569507840(7n2−43n−23)−(219240n−1466005)(50112n+27256)=3858751960∣3858751960
The prime factorisation of 3858751960 is 23⋅5⋅7⋅1968751 (you can check the last term is a prime though any means of primality checking).
Since n5+5 and (n+1)5+5 are alternatively odd and even, 2 cannot be their common factor. Also, by FLT, since n5≡n(mod5), d cannot be equal to 5. By inspection, it can be seen that n5 is not congruent to (n+1)5+5 in (mod7). Thus, d=1 or d=1968751.
We will now find when the gcd(n5+5,(n+1)5+5)>1.
Note from above that d must be a common divisor of both 50112n+27256 and 1968751. Since 1968751 is a prime, if we have 50112n+27256≡0(mod1968751), we must then have that d=1.
Thus, we have 50112n≡1941495(mod1968751). We can solve this by finding the modular inverse of 50112. Since 50112 has small factors, we needn't apply Euclid's algorithm. We instead use inspection to find the inverse.
50112=26⋅33⋅29. We will now check the value of each factor's inverse in (mod1968751).
2×984376⇒2−1⇒2−3⇒2−6=1968751+1≡984376(mod1968751)≡984376÷4=246094(mod1968751)≡2460942≡1507325(mod1968751)
Similarly, 3−3≡1239584(mod1968751) and 29−1≡67888. Thus, the modular inverse of 50112 is
50112−1≡2−6⋅3−3⋅29−1≡1507325×1239584×67888≡1731811(mod1968751)
Thus, we have
50112nn≡1941495(mod1968751)≡194195×1731811≡533360(mod1968751)
Therefore, when n≡533360(mod1968751), n5+5 and (n+1)5+5 are not coprime.
This problem seems difficult than it looks... I have done half of the proof , just trll me no casework is required isnt it ? At first I tried to provethat by a little Modular Arith. but I am getting stuck in between...I am missing something basic...btw sharky you can arite the solution for shivams problem...This problem has made me crazy...@Sharky Kesa
Log in to reply
I didn't use casework in my solution, mainly just divisibility and congruences. Also, because no one has yet given a solution, I will say that there are values of n such that gcd(n5+5,(n+1)5+5)>1. @Chinmay Sangawadekar @Alex Spagnoletti
Log in to reply
d∣a and d∣b, then d∣(ax+by)).
Hint 2: You will want to know that 1968751 is a prime number if you wish to solve this. Also, use mainly divisibility laws (ifSolution of problem "extra partecipation problem -set 2: #2" n5+5 and (n+1)5+5 have no common divisors because seen that n5 and (n+1)5 are coprime for all positive integers n if we add 5 to both they will remain coprime.
Log in to reply
@Alex Spagnoletti That isn't true.Counterexample: gcd(1,3)=1 but gcd(1+5,3+5)=gcd(6,8)=2=1
Every pair of odd positive integers a,b with gcd(a,b)=1 is a counterexample,as gcd(a+5,b+5)≥2 as both a+5 and b+5 are even numbers.
Log in to reply
@Abdur Rehman Zahid, Yes but 3 is not 1+1 It must be n and n+1
This is not a correct solution. Just because two numbers are coprime doesnt mean that a constant added to both numbers will keep them both coprime.
Log in to reply
@Sharky Kesa It is not true for all numbers. But for n and n+1 this is true. For example an even number can't be the gcd because they are one even and one odd. A 3k number can't be the gcd because if n is a 3k number 3k + 5 is not a 3k number and n and n+1 can be either one 3k and the other 3k+1( or 3k-1) or they can be both non 3k numbers. In this case adding 5 will make one 3k and the other not. For 5k numbers it's easy to see that if n is 5k, n+5 is also 5k but n+1 is not 5k and so it can't be the gcd. So it's the same for the other numbers and naturally this is the same with n5 and (n+1)5 seen that they are just n and (n+1) 5 times. We can also think to prove this absurdly. If n+5 and (n+1)+5 are not coprime this means that gcd∣n+5 and gcd∣n+6 but there is no number > 1 that can divide both a number and that number + 1 I think that this reasoning has no mistakes, but if I have made any mistakes please tell me because I can't find them.
Log in to reply
Log in to reply
@Sharky Kesa It's right. But n5 and (n+1)5 are divisibile by n and (n+1) so it is the same. It is like if they are separated but they are always divisible by 2 consecutive numbers. So working modulo n I will have 0+5 and 1+5. So they are consecutive in this form. (And there is another time the absurd)
Log in to reply
Log in to reply
Log in to reply
Alternate Solution to Problem 5:
Recursion was my strategy.
Let S(N)=q=1∑nd2q−1⋅d2q where N and di are as defined.
Let f(n) denote the number of ways we can find a suitable N of 2n digits, that satisfies S(N)≡0 mod 2.
We need to find a recurrence for f(n).
Now consider N0, which is a number with 2n−2 digits.
Now, if N0 has S(N) even, then we need to find d2n−1,d2n such that their product is even.
Note that the number of N0 here is f(n−1).
The number of ways to find such d2n−1,d2n is f(1)=56, by simple casework.
If N0 has S(N) odd, then we need to find d2n−1,d2nsuch that their product is odd.
The number of the number of N0 here is all the N0 not counted in the case above.
As there are 92n−2 possible numbers totally, and each number must fall in either of these categories, there are 92n−2−f(n−1) such N0 here.
The number of ways to find such d2n−1,d2n is 92−f(1)=25.
Then, f(n)=56⋅f(n−1)+25⋅(92n−2−f(n−1)) for n>1, and f(1)=56
⇒f(n)=31⋅f(n−1)+25⋅92n−2
With some juggling,
f(n)=31⋅f(n−1)+281n−31⋅81n−1
Noticing that, f(1)=56=281+31,
We are motivated to try, f(n)=281n+31n which satisfies the recurrence as well.
∴f(n)=281n+31n
Extra Participation Problem Set - 2 - #3
Let n be an integer greater than 6. Prove that if n−1 and n+1 are both prime then n2(n2+16) is divisible by 720.
Bonus Prove that the converse is not true.
Log in to reply
Being all the primes >5 ends with 1,3,7 or 9 n will end with 0,8 or 2 because it must be in the middle of two primes. So 2∣nand 3∣n* for all n means that 22∣n2and 32∣n2. But seen that n is either 5k (if it ends with 0) or it is ≡2,8(mod10) n2 will be either 5k or ≡4(mod10). So adding 16 will make n2≡0(mod10) and so 5∣n or in the case of n=5k it is already ≡0(mod10). 720=24×32×5 so n2(n2+16) can be divided in 2 cases:
1) if n2=22×32×5×? and (n2+16)=22×?=>n2(n2+16)=24×32×5×? and so it is divisible by 720.
2) if n2=22×32×? and (n2+16)=22×5×?=>n2(n2+16)=24×32×5×? and so it is divisible by 720.
Log in to reply
Correct! Here is my approach
Any prime number>3 is of the form 1,5(mod6). Thus n is divisible by 6 take n as 6m.
Thus it suffices to prove that m2(9m2+4) is divisible by 5.
We eliminate the case m≡1,4(mod5) because then n−1 and n+1 will be divisible by 5 respectively(so it won't be prime then).
Checking the cases m≡0,2,3(mod5) we see that each of the cases satisfies.
Hence proved.
Bonus: Take n=1440 the expression is divisible by 720 but 1441 is not prime (11×131)
Log in to reply
n=258 makes the expression dibisible by 720 but 259 is not prime. If n is divisible by 6 and it ends with 8 , 4 or 0 the expression is satisfied also if (n+1) or (n−1) are not prime.
It is a shorter proof, I like it. However alsoLog in to reply
Log in to reply
Log in to reply
Log in to reply
Problem 8
Let a, b, x and y be positive integers, with x>1. Prove that if ax+bx=2y, then a=b.
This question is based off AMO 2016 Q6, but a more general form.
Log in to reply
Let a≥b, then we have ax≥bx or ax+bx≥2bx
This means 2y−1≥bx
Now consider b≥a then we have bx≥ax or 2bx≥ax+bx
This means bx≥2y−1
Therefore bx=2y−1
Similarly we can prove ax=2y−1.
Hence we have ax=bx⟹a=b
How much will I get @Sharky Kesa
Log in to reply
You have proven the converse of the problem statement. In the first case, you have proven 2y−1≥bx if a≥b. In the second case, you have proven 2y−1≤bx if a≤b. From this, you are saying that if a=b, then bx=2y−1. Similarly, you are saying that ax=2y−1, from which you get ax+bx=2y if a=b (This is incorrect of course). In any case, you are trying to prove the converse of the problem statement.
Problem 3
Prove that
d∣n∑μ(n)={1 if n=10 if n>1
Not original.
This problem has been solved by Akshat Sharda
Log in to reply
Solution to Problem 3
This is already a well known problem.
Let,
n=p1a1⋅p2a2⋅p3a3⋅…⋅prar
As d∣n,
d=p1b1⋅p2b2⋅p3b3⋅…⋅prbr
Here, 0≤bi≤ai,i=1,2,…,r. If bi≥2,μ(d)=0. Therefore,
\begin{aligned} \displaystyle \sum_{d|n} \mu(d) & =\displaystyle \sum_{\substack{(b_1,b_2, \ldots , b_r) \\ b_i = 0 \text{ or } 1}} \mu( p_{1}^{b_1}\cdot p_{2}^{b_2}\cdot p_{3}^{b_3}\cdot \ldots \cdot p_{r}^{b_r}) \\ &=1 - \binom{r}{1} + \binom{r}{2} - \cdots + (-1)^{r} \binom {r}{r} \\ &= (1 - 1)^{r} \\ &= 0 \end{aligned}
Problem 4
Let {Xn} and {Yn} denote 2 sequences of integers as follows:
X0=X1=1,Xn+1=Xn+2Xn−1Y0=1,Y1=7,Yn+1=2Yn+3Yn−1
Prove that except for 1, there is no other term which occur in both sequences.
Not original.
This problem has been solved by Shivam Jadhav.
Log in to reply
Answer to Problem 4: Using concept of linear recurrence relations we get xn=32n+1+(−1)n,yn=2(3)n+(−1)n+1
Now let xm=yn
2(3n+1−2m)=3(−1)m+(−1)n RHS=4,−4,2,−2 Therefore
(3n+1−2m) can be equal to 2,−2,1,−1
But LHS=odd Since 2w=even,3q=odd and odd−even=odd.
Therefore (3n+1−2m)=1,−1 which is possible only for (m,n)=(0,1) while taking the RHS in consideration . Therefore only x0=y1 Hence proved .
Extra Participation Problem - Set 1 - #1
Show that for every positive integer n,
121n−25n+1900n−(−4)n is divisible by 2000.
This problem has been solved by Ankit Kumar Jain and Akshat Sharda.
Log in to reply
@Svatejas Shivakumar My solution is this:
What I will be doing is that I show that the expression is divisible by 125 and also by 16.
121n−25n is divisible by 121 - 25 = 96 and hence by 16. Also 1900n−(−4)n is divisible by 1900 - (-4) = 1904 and hence by 16. So the expression is divisible by 16.
1900n−25n is divisible by 1900 - 25 = 1875 and hence by 125. Also 121n−(−4)n is divisible by 121 - (-4) = 125. So the whole expression is divisible by 125.
Hence the expression is divisible by 125×16...
As a shortcut you can straight away put n = 1 and check....
121n−25n+1900n−(−4)n(mod2)1n−1n+0−0≡0(mod2)121n−25n+1900n−(−4)n(mod5)n= odd1n+0+0−(−4)≡0(mod5)n= even1n+0+0−(6)≡0(mod5)
Therefore, 121n−25n+1900n−(−4)n≡0(mod2000)
Log in to reply
Can you please explain how you concluded that it is divisible by 2000(last step)?
Log in to reply
Extra Participation Problem - Set 1 - #4
Prove that (n+1)(n+2)…(2n−1)(2n) is divisible by 2n.
Log in to reply
I think there's a relatively simpler argument.
For all positive integers n, we have,
(n+1)(n+2)⋯(2n−1)(2n)=n!(2n)!=n!(2n)!!(2n−1)!!=n!2n⋅n!⋅(2n−1)!!=2n(2n−1)!!
which is obviously divisible by 2n for all n∈Z+ since (2n−1)!! is an integer for all n∈Z+.
Notation: n!! denotes the double factorial.
Solution to Extra Participation Problem-Set 1-#4
Here , we know that , (n+1)(n+2)...(2n−1)(2n)≡0mod2 , now multiplying the whole congruence by ,
2n−1 , we get , 2n−1(n+1)(n+2)...(2n−1)(2n)≡0mod2n , so from here , we know that , 2n
does not divide 2n−1 but still remainder is 0 , so it is obvious that , 2n∣(n+1)(n+2)...(2n−1)(2n)
Log in to reply
How do you conclude that since 2n∣2n−1, we must have 2n∣i=1∏n(n+i) ? Consider a=24 , b=23 , c=2, we have a∣bc and a∣b but that doesn't imply a∣c.
Extra Participation Problem- Set 1 - #6
Prove that when n∈Z+,n=1, and 2n+n2 is prime, then n≡3(mod6).
This problem is not original.
This problem has been solved by Svatejas Shivakumar.
Log in to reply
Solution to Extra Participation Problem - Set 1 - #6
If a numbers is a prime number >3, it is ≡1,5(mod6)
So, 2n+n2≡1,5(mod6) for n=1
If n≡0(mod6), 2n+n2≡0(mod6)
If n≡1(mod6), 2n+n2≡3(mod6)
If n≡2(mod6), 2n+n2≡2(mod6)
If n≡3(mod6), 2n+n2≡5(mod6)
If n≡4(mod6), 2n+n2≡2(mod6)
If n≡5(mod6), 2n+n2≡3(mod6)
Only the fourth case is possible.
Hence if 2n+n2 is prime, then n≡3(mod6)
Extra Participation Problem - set 2 -#1
Prove that 9∣((7n+1)n−2n)3 ,for all positive integers n
This problem has been solved by Sharky K
Log in to reply
7n+12n−2n≡1n+1(mod3)≡2≡0
Thus, 3∣(7n+1)n−2n, which implies 27∣((7n+1)n−2n)3, so 9 divides the expression. Thus proven.
Log in to reply
Yep !
Problem 7
Which positive integers can be written in the form
x2+y2−z2
for positive integers 1≤x<y<z?
Log in to reply
My solution:
I want to prove that all positive integers can be expressed in this form by giving a construction. Let m be the positive integer which is expressed in this form. We have
x2+y2−z2x2−m=m=(z+y)(z−y)
Let z−y=1. We then must have z+y=x2−m. Solving this pair of linear equations, we get y=2x2−m−1 and z=2x2−m+1. We now want both numerators to be even for all values of m by finding a suitable expression for x in terms of m. If we let x=m+k (for some odd number k), this would satisfy the numerators being even for all values of x. However, we must also satisfy that 1≤x<y<z. However, we already have y<z. Let us substitute this expression to find a bound for k.
xm+k2m+2k2m<y<2(m+k)2−k−1<m2+2km+k2−k−1<m2+2km+k2−3k−1
Notice how we want to keep k2−3k−1 positive? Thus, we must have k>3. The smallest odd value that satisfies is k=5. We can use this for our construction. Thus, our construction is
xyz=m+5=2m2+9m+24=2m2+9m+26
Verifying, we get
x2+y2−z2(m+5)2+4(m2+9m+24)2−4(m2+9m+26)2m2+10m+25−(m2+9m+25)m=m=m=m=m
Thus, our construction is finished and we have proved that all positive integer can be formed using this construction.
The positive integers that can be written in the form x2+y2−z2 are those that satisfies this progression
xn=xn−1+7+2(n−1)x0=5
So if a=(n+1)(n+5) a can be written in the form x2+y2−z2
However probably there are also other numbers, but there are infinite numbers.
Log in to reply
Exactly same approach I had in my mind...Nice solution ...:) :)
Log in to reply
This is not a complete solution, but a good try. Try again, you are close to the answer.
Log in to reply
x2+y2−z2 also infinite numbers can be generated. And generalizinga=(n+1)(n+5) I wrote a=n(n+α) where α>−n. With this every positive number can be generated. Now I'm not sure of this generalization, but at the moment is the only explaination I can find.
After some reasoning I realized that seen that there are infinite combinations ofLog in to reply
Log in to reply
Log in to reply
Log in to reply
Please post the solution.
Extra Participation Problem - Set 1 - #2
For any integer n≥3 , prove that , k=1∑nμ(k!)=1
This problem has been solved by Akshat.S
Log in to reply
For k≥4,k! would be divisible by 22, thus μ(k!)=0.
Therefore,
k=1∑nμ(k!),∀n≥3=k=1∑3μ(k!)=μ(1)+μ(2)+μ(3)=1+(−1)1+(−1)1=1
Extra Participation Problem-Set 1 - #5
Show that , 1000! terminates in 249 zeroes.
Log in to reply
Solution to Extra Participation Problem - Set 1 - #5
The formula for trailing zeroes is: i=1∑k⌊5in⌋, such that 5k+1>n.
55=3125>1000⟹k=4.
⌊541000⌋+⌊531000⌋+⌊521000⌋+⌊511000⌋=1+8+40+200=249.
Extra Participation Problem Set - 2 - #4
Prove that if x2≡a(modp) with p prime and >3 there are only 2 solutions which are s+r=p.
This problem has been solved by Sharky Kesa.
Log in to reply
What is s and r and what are we supposed to solve?
Log in to reply
You have to prove that for each a there are only 2 solutions which are s+r=p. It is a quadratic residue so for some a and p there could be no solutions but we are assuming that there are solutions. Ex. 32≡4(mod5) and 22≡4(mod5) so 3+2=5.
So why the solutions are r and s=p−r and prove that there are only this 2 solutions.
I assume we also have 0<x<p? Otherwise, your statement is incorrect.
We will prove that if x2≡a(modp), then (p−x)2≡a(modp) (By proving this, we get that the sum of the two values is p.
Let x2≡a(modp). We have
(p−x)2≡p2−2px+x2≡x2≡a(modp)(modp)(modp)
Thus, we have that if there are 2 solutions, then their sum is indeed p and the two solutions are s=x and r=p−x. We will now prove that there are only 2 solutions. Let t2≡a(modp) where t=x,p−x and 0<t<p. We have
t2x2−t2(x−t)(x+t)≡a(modp)≡0(modp)≡0(modp)
Thus, either p∣x−t or p∣x+t. If p∣x−t and 0<x,t<p, we have that x−t=0, which implies x=t, which contradicts our first statement that t=x. If p∣x+t and 0<x,t<p, we have that x+t=p, which implies t=p−x, which contradicts our first statement that t=p−x. Thus, no such t exists.
We have proven there exist only 2 values for x such that x2≡a(modp), and that the sum of these two values is p.
Log in to reply
Yes, it's exactly my proof. Obviously seen that the 2 solutions are s+r=p they are less than p.
Problem 5
Let N be a 2n digit number with digits d1,d2,d3,...,d2n from left to right i.e N=d1d2....d2n where dp is not equal to 0 , p=1,2,...,2n. Find the number of such N so that the sum q=1∑nd2q−1×d2q=even in terms of n .
Log in to reply
It seems more like a combinatorics problem.
Log in to reply
Number theory + Combinatorics
First Note that
q=1∑nd2q−1×d2q=2k
So clearly that when N is taken in groups of 2 like d1×d2,d3×d4,.... this means at least one of them is even in the partition so at least n of them is even.
Consider when n=1:
Let the first digit ( d1 ) be even and the other be from 1 to 9, so we get total possibility of 4×9.
Let the second digit be ( d2 ) be even then we again get a total possibility of 4×9. Adding them we get 72=(41×91)×2. But if we carefully look we will find out that we have counted an extra case of number when all digits are same and even. Since we are looking numbers ranging from 11 to 99 in this case we have to find all numbers that have same digit and are even.
This means it should be the multiple of 22 hence total numbers are ⌊2299⌋=4. So total cases are 72−4=68=2(361×1−2)
Similarly for
n=2 we get 2(362×2−2)
n=3 we get 2(363×3−2).
At the end we see the general pattern is 2(36n×n−2).
Extra Participation Problem Set - 3 - #1
Find the smallest value of n such that the arithmetic mean of the sequence 12,22,…,n2 is itself a perfect square.
Log in to reply
Using the Sum of Squares formula, we have
6nn(n+1)(2n+1)2n2+3n+1nn=k2=6k2=2−3±9+48k2−8=2−3±48k2+1
We need to find the smallest value of k such that 48k2+1 is a perfect square, an k>1. This is just a Pell's equation, which has solutions for
48k2+1=x2
as
(x,k)=(7,1),(97,14),(1351,195),…
The second solution gives fractional n and the third solution gives n=337. Thus, n=337 is the smallest solution >1.
I've already done this exercise on brilliant. The answer is 337 if I remember well.
Extra Participation Problem Set 3: #2
In how many ways can 50! be written as the sum of 2 or more consecutive numbers?
Log in to reply
We should have 50!=nx+2n(n−1) where n>2 is the number of consecutive numbers and x is the first number.
So 2n2×50!+n(1−n)=x so n∣50! and 2∣1−n.
So we have to find the number of odd numbers that divides 50!
The prime(except 2) factorization of 50! is 322×512×78×114×133×172×192×232×29×31×37×41×43×47
So the number of odd numbers that divides 50! is 23×13×9×5×4×3×3×3×2×2×2×2×2×2=93000960
Log in to reply
Set 2 - Problem 5. Did we finish that set?
Extra Participation Problem Set 3: #3
Let m=34p−1, where p is a prime number exceeding 3. Prove that 2m−1 has remainder 1 when divided by m.
Log in to reply
I think I have found something. First we can notice that m−1 is always divisible by a certain k which is 2 times p. Then the ordm(2) is k so 2k×h≡1(modm). Being m−1divisible by k 2m−1 will be ≡1(modm).
Log in to reply
First I thought if we prove m is prime then this problem gets proved but that was wrong...later I tried something to do with parity of m ' then rearrangements of congruences etc....but still not able to get the key operation :( have you gotanything else ?
Log in to reply
2p and that 2p is also the ordm(2)
No :( the only thing is that m-1 is divisible byLog in to reply
Log in to reply
ordm(2) is 2p
Yes, I have just to find a formal proof because at the moment I don't know why theLog in to reply
Log in to reply
Log in to reply
Log in to reply
Svatejas's problem is still unsolved.... May be we should discuss about it in next season...
Log in to reply
Yes, I think it's better